My answer don't match with the given answer.

Win_odd Dhamnekar

Junior Member
Joined
Aug 14, 2018
Messages
207
A vessel A is a sailing with a velocity of 11 knots per hour in the direction south-east and a second vessel B is sailing with a velocity of 13 knots per hour in a direction 30° of north. Find the velocity of A relative to B.

Answer: \(\displaystyle v_A=\left[\frac{11}{2}, -\frac{11}{2}\right], v_B =\left[ -\frac{13}{2}, \frac{13\sqrt{3}}{2}\right], v_{A-B}=[14.28,-19.04 ]=23.80 \) knots/hr.

Author gave answer 21.29 knots/hr in the direction 74° 47' south of east. How is that computed?
 
A vessel A is a sailing with a velocity of 11 knots per hour in the direction south-east and a second vessel B is sailing with a velocity of 13 knots per hour in a direction 30° of north. Find the velocity of A relative to B.

Answer: \(\displaystyle v_A=\left[\frac{11}{2}, -\frac{11}{2}\right], v_B =\left[ -\frac{13}{2}, \frac{13\sqrt{3}}{2}\right], v_{A-B}=[14.28,-19.04 ]=23.80 \) knots/hr.

Author gave answer 21.29 knots/hr in the direction 74° 47' south of east. How is that computed?
First, though this doesn't affect the work, a "knot per hour" would be an acceleration, not a velocity: "knot" means "nautical mile per hour", not a distance. (But see here.)

Also, when the question says "30° of north", I would take it to mean "30° east of north" (if it is not just an error), not as you apparently did, "30° west of north". But perhaps you omitted the word "west" when you copied the problem?

But your main error is that the magnitude of your vA is not 11. Check that!
 
First, though this doesn't affect the work, a "knot per hour" would be an acceleration, not a velocity: "knot" means "nautical mile per hour", not a distance. (But see here.)

Also, when the question says "30° of north", I would take it to mean "30° east of north" (if it is not just an error), not as you apparently did, "30° west of north". But perhaps you omitted the word "west" when you copied the problem?

But your main error is that the magnitude of your vA is not 11. Check that!
\(\displaystyle v_A\) is a typographical error. It should be \(\displaystyle v_A= \left[\frac{11}{\sqrt{2}},-\frac{11}{\sqrt{2}}\right]\) If we take 30° east of north, then \(\displaystyle v_B=\left[\frac{13}{2}, -\frac{13\sqrt{3}}{2}\right]\)

Now \(\displaystyle v_{A-B}= [1.28, 3.48], ABS(v_{A-B})=3.707\) knots/hr.

That means even after adjusting \(\displaystyle v_B\) I don't get author's answer. How is that? is answer given wrong?
 
\(\displaystyle v_A\) is a typographical error. It should be \(\displaystyle v_A= \left[\frac{11}{\sqrt{2}},-\frac{11}{\sqrt{2}}\right]\) If we take 30° east of north, then \(\displaystyle v_B=\left[\frac{13}{2}, -\frac{13\sqrt{3}}{2}\right]\)

Now \(\displaystyle v_{A-B}= [1.28, 3.48], ABS(v_{A-B})=3.707\) knots/hr.

That means even after adjusting \(\displaystyle v_B\) I don't get author's answer. How is that? is answer given wrong?
You still have a wrong sign in B.

But I haven't found a way to get their answer, either. Can you confirm the exact wording of the problem (particularly the direction of B), perhaps as an image?
 
You still have a wrong sign in B.

But I haven't found a way to get their answer, either. Can you confirm the exact wording of the problem (particularly the direction of B), perhaps as an image?
\(\displaystyle v_B\) describes \(\displaystyle \frac{13}{2}\hat{i}\) for 30° east of North and \(\displaystyle -\frac{13\sqrt{3}}{2}\hat{j}\) describe 60° north of east. Is that incorrect?
 
\(\displaystyle v_B\) describes \(\displaystyle \frac{13}{2}\hat{i}\) for 30° east of North and \(\displaystyle -\frac{13\sqrt{3}}{2}\hat{j}\) describe 60° north of east. Is that incorrect?
Why is north negative?

I recommend drawing a fairly accurate figure to check your work and your answers, particularly since the book appears to be wrong.
 
Why is north negative?

I recommend drawing a fairly accurate figure to check your work and your answers, particularly since the book appears to be wrong.
Okay, We take north of east \(\displaystyle \frac{13\sqrt{3}}{2}\hat{j}\) positive in \(\displaystyle v_B\). In \(\displaystyle v_A, \frac{11}{\sqrt{2}}\hat{i}\) is 45° east of south and \(\displaystyle -\frac{11}{\sqrt{2}}\hat{j}\) is 45°south of east. Is this correct?

Why is it wrong to take \(\displaystyle \frac{11}{\sqrt{2}}\hat{i}\) 45° east of south negative in \(\displaystyle v_A?\)

Is it just because we took \(\displaystyle \frac{13}{2}\hat{i}\) in \(\displaystyle v_A\) positive?
 
Okay, We take north of east \(\displaystyle \frac{13\sqrt{3}}{2}\hat{j}\) positive in \(\displaystyle v_B\). In \(\displaystyle v_A, \frac{11}{\sqrt{2}}\hat{i}\) is 45° east of south and \(\displaystyle -\frac{11}{\sqrt{2}}\hat{j}\) is 45°south of east. Is this correct?

Why is it wrong to take \(\displaystyle \frac{11}{\sqrt{2}}\hat{i}\) 45° east of south negative in \(\displaystyle v_A?\)

Is it just because we took \(\displaystyle \frac{13}{2}\hat{i}\) in \(\displaystyle v_A\) positive?
Did I say that both were wrong? Only the one. I think I only said,
You still have a wrong sign in B.
Why is north negative?

But, again, I haven't found a way to get their answer by changing any one thing, so I wouldn't worry about not having their answer. Just draw a picture and see if your answer matches that.
 
Top